summaryrefslogtreecommitdiff
path: root/source/know/concept/convolution-theorem/index.md
diff options
context:
space:
mode:
Diffstat (limited to 'source/know/concept/convolution-theorem/index.md')
-rw-r--r--source/know/concept/convolution-theorem/index.md77
1 files changed, 44 insertions, 33 deletions
diff --git a/source/know/concept/convolution-theorem/index.md b/source/know/concept/convolution-theorem/index.md
index d10d85d..3f9eafb 100644
--- a/source/know/concept/convolution-theorem/index.md
+++ b/source/know/concept/convolution-theorem/index.md
@@ -7,54 +7,59 @@ categories:
layout: "concept"
---
-The **convolution theorem** states that a convolution in the direct domain
-is equal to a product in the frequency domain. This is especially useful
-for computation, replacing an $$\mathcal{O}(n^2)$$ convolution with an
-$$\mathcal{O}(n \log(n))$$ transform and product.
+The **convolution theorem** states that a convolution in the real domain
+is equal to a product in the frequency domain.
+This fact is especially useful for computation,
+as it allows replacing an $$\mathcal{O}(n^2)$$ convolution
+with an $$\mathcal{O}(n \log(n))$$ transform and product.
## Fourier transform
-The convolution theorem is usually expressed as follows, where
-$$\hat{\mathcal{F}}$$ is the [Fourier transform](/know/concept/fourier-transform/),
-and $$A$$ and $$B$$ are constants from its definition:
+The convolution theorem is usually expressed as follows,
+where $$\hat{\mathcal{F}}$$ is the [Fourier transform](/know/concept/fourier-transform/),
+and $$A$$ and $$B$$ are the constants from its definition:
$$\begin{aligned}
\boxed{
\begin{aligned}
- A \cdot (f * g)(x) &= \hat{\mathcal{F}}{}^{-1}\{\tilde{f}(k) \: \tilde{g}(k)\} \\
- B \cdot (\tilde{f} * \tilde{g})(k) &= \hat{\mathcal{F}}\{f(x) \: g(x)\}
+ A \cdot (f * g)(x)
+ &= \hat{\mathcal{F}}{}^{-1}\Big\{ \tilde{f}(k) \: \tilde{g}(k) \Big\}
+ \\
+ B \cdot (\tilde{f} * \tilde{g})(k)
+ &= \hat{\mathcal{F}}\Big\{ f(x) \: g(x) \Big\}
\end{aligned}
}
\end{aligned}$$
{% include proof/start.html id="proof-fourier" -%}
-We expand the right-hand side of the theorem and
-rearrange the integrals:
+We expand the right-hand side of the theorem and rearrange the integrals:
$$\begin{aligned}
- \hat{\mathcal{F}}{}^{-1}\{\tilde{f}(k) \: \tilde{g}(k)\}
- &= B \int_{-\infty}^\infty \tilde{f}(k) \Big( A \int_{-\infty}^\infty g(x') \: e^{i s k x'} \dd{x'} \Big) e^{-i s k x} \dd{k}
+ \hat{\mathcal{F}}{}^{-1}\Big\{ \tilde{f}(k) \: \tilde{g}(k) \Big\}
+ &= B \int_{-\infty}^\infty \tilde{f}(k) \bigg( A \int_{-\infty}^\infty g(x') \: e^{i s k x'} \dd{x'} \bigg) e^{-i s k x} \dd{k}
\\
- &= A \int_{-\infty}^\infty g(x') \Big( B \int_{-\infty}^\infty \tilde{f}(k) \: e^{-i s k (x - x')} \dd{k} \Big) \dd{x'}
+ &= A \int_{-\infty}^\infty g(x') \bigg( B \int_{-\infty}^\infty \tilde{f}(k) \: e^{-i s k (x - x')} \dd{k} \bigg) \dd{x'}
\\
&= A \int_{-\infty}^\infty g(x') \: f(x - x') \dd{x'}
- = A \cdot (f * g)(x)
+ \\
+ &= A \cdot (f * g)(x)
\end{aligned}$$
Then we do the same again,
this time starting from a product in the $$x$$-domain:
$$\begin{aligned}
- \hat{\mathcal{F}}\{f(x) \: g(x)\}
- &= A \int_{-\infty}^\infty f(x) \Big( B \int_{-\infty}^\infty \tilde{g}(k') \: e^{-i s x k'} \dd{k'} \Big) e^{i s k x} \dd{x}
+ \hat{\mathcal{F}}\Big\{ f(x) \: g(x) \Big\}
+ &= A \int_{-\infty}^\infty f(x) \bigg( B \int_{-\infty}^\infty \tilde{g}(k') \: e^{-i s x k'} \dd{k'} \bigg) e^{i s k x} \dd{x}
\\
- &= B \int_{-\infty}^\infty \tilde{g}(k') \Big( A \int_{-\infty}^\infty f(x) \: e^{i s x (k - k')} \dd{x} \Big) \dd{k'}
+ &= B \int_{-\infty}^\infty \tilde{g}(k') \bigg( A \int_{-\infty}^\infty f(x) \: e^{i s x (k - k')} \dd{x} \bigg) \dd{k'}
\\
&= B \int_{-\infty}^\infty \tilde{g}(k') \: \tilde{f}(k - k') \dd{k'}
- = B \cdot (\tilde{f} * \tilde{g})(k)
+ \\
+ &= B \cdot (\tilde{f} * \tilde{g})(k)
\end{aligned}$$
{% include proof/end.html id="proof-fourier" %}
@@ -62,45 +67,51 @@ $$\begin{aligned}
## Laplace transform
-For functions $$f(t)$$ and $$g(t)$$ which are only defined for $$t \ge 0$$,
+For functions $$f(t)$$ and $$g(t)$$ that are only defined for $$t \ge 0$$,
the convolution theorem can also be stated using
the [Laplace transform](/know/concept/laplace-transform/):
$$\begin{aligned}
- \boxed{(f * g)(t) = \hat{\mathcal{L}}{}^{-1}\{\tilde{f}(s) \: \tilde{g}(s)\}}
+ \boxed{
+ (f * g)(t)
+ = \hat{\mathcal{L}}{}^{-1}\Big\{ \tilde{f}(s) \: \tilde{g}(s) \Big\}
+ }
\end{aligned}$$
-Because the inverse Laplace transform $$\hat{\mathcal{L}}{}^{-1}$$ is
-unpleasant, the theorem is often stated using the forward transform
-instead:
+Because the inverse Laplace transform $$\hat{\mathcal{L}}{}^{-1}$$ is usually difficult,
+the theorem is often stated using the forward transform instead:
$$\begin{aligned}
- \boxed{\hat{\mathcal{L}}\{(f * g)(t)\} = \tilde{f}(s) \: \tilde{g}(s)}
+ \boxed{
+ \hat{\mathcal{L}}\Big\{ (f * g)(t) \Big\}
+ = \tilde{f}(s) \: \tilde{g}(s)
+ }
\end{aligned}$$
{% include proof/start.html id="proof-laplace" -%}
We expand the left-hand side.
Note that the lower integration limit is 0 instead of $$-\infty$$,
-because we set both $$f(t)$$ and $$g(t)$$ to zero for $$t < 0$$:
+because we choose to set both $$f(t)$$ and $$g(t)$$ to zero for $$t < 0$$:
$$\begin{aligned}
- \hat{\mathcal{L}}\{(f * g)(t)\}
- &= \int_0^\infty \Big( \int_0^\infty g(t') \: f(t - t') \dd{t'} \Big) e^{-s t} \dd{t}
+ \hat{\mathcal{L}}\Big\{ (f * g)(t) \Big\}
+ &= \int_0^\infty \bigg( \int_0^\infty g(t') \: f(t - t') \dd{t'} \bigg) e^{-s t} \dd{t}
\\
- &= \int_0^\infty \Big( \int_0^\infty f(t - t') \: e^{-s t} \dd{t} \Big) g(t') \dd{t'}
+ &= \int_0^\infty \bigg( \int_0^\infty f(t - t') \: e^{-s t} \dd{t} \bigg) \: g(t') \dd{t'}
\end{aligned}$$
Then we define a new integration variable $$\tau = t - t'$$, yielding:
$$\begin{aligned}
- \hat{\mathcal{L}}\{(f * g)(t)\}
- &= \int_0^\infty \Big( \int_0^\infty f(\tau) \: e^{-s (\tau + t')} \dd{\tau} \Big) g(t') \dd{t'}
+ \hat{\mathcal{L}}\Big\{ (f * g)(t) \Big\}
+ &= \int_0^\infty \bigg( \int_0^\infty f(\tau) \: e^{-s (\tau + t')} \dd{\tau} \bigg) \: g(t') \dd{t'}
\\
- &= \int_0^\infty \Big( \int_0^\infty f(\tau) \: e^{-s \tau} \dd{\tau} \Big) g(t') \: e^{-s t'} \dd{t'}
+ &= \int_0^\infty \bigg( \int_0^\infty f(\tau) \: e^{-s \tau} \dd{\tau} \bigg) \: g(t') \: e^{-s t'} \dd{t'}
\\
&= \int_0^\infty \tilde{f}(s) \: g(t') \: e^{-s t'} \dd{t'}
- = \tilde{f}(s) \: \tilde{g}(s)
+ \\
+ &= \tilde{f}(s) \: \tilde{g}(s)
\end{aligned}$$
{% include proof/end.html id="proof-laplace" %}